$liminf E_{n_0+n} = liminflimits E_{n}$ and $limsup E_{n_0+n} = limsup E_{n}$ where $E_n$ is a decreasing...












1














I dropped the $nrightarrowinfty$ in the title as it was exceeding the character limit.



In the book I'm currently reading, the author claims that $liminflimits_{nrightarrowinfty} E_{n_0+n} = liminflimits_{nrightarrowinfty} E_{n}$ and $limsuplimits_{nrightarrowinfty} E_{n_0+n} = limsuplimits_{nrightarrowinfty} E_{n}$ where $mu(E_{n_{0}}) < infty$ and $E_n$ is a decreasing sequence? ($mu$ is an arbitrary measure!)





My attempt at a proof is as follows:



Let $x in liminflimits_{nrightarrowinfty} E_{n_0+n} = bigcup_{ninmathbb{N}}bigcap_{kgeq n}E_{n_0+n}$



Then $x in bigcap_{kgeq n}E_{n_0+n_{0}'}$ for some $n_{0}' in mathbb{N}$



But $bigcap_{kgeq n}E_{n_0+n_{0}'} subset bigcup_{ninmathbb{N}}bigcap_{kgeq n}E_{n} = liminflimits_{nrightarrowinfty} E_{n}$ and so $liminflimits_{nrightarrowinfty} E_{n_0+n} subset liminflimits_{nrightarrowinfty} E_{n}$.



I however couldn't prove the reverse inclusion in my attempt below:



Let $x in liminflimits_{nrightarrowinfty} E_{n} = bigcup_{ninmathbb{N}}bigcap_{kgeq n}E_{n}$



Then $x in bigcap_{kgeq n}E_{n_{0}''}$ for some $n_{0}'' in mathbb{N}$



I then realized it's not guaranteed that $n_{0}'' = n_{0}$ and so we may be "missing" some elements in $bigcup_{ninmathbb{N}}bigcap_{kgeq n}E_{n_0+n}$ from $bigcup_{ninmathbb{N}}bigcap_{kgeq n}E_{n}$.



I suspect the proof for $limsuplimits_{nrightarrowinfty} E_{n_0+n} = limsuplimits_{nrightarrowinfty} E_{n}$ is similar and would also fall apart at the same point as my proof for $liminflimits_{nrightarrowinfty} E_{n_0+n} = liminflimits_{nrightarrowinfty} E_{n}$ ($n_{0} neq n_{0}''$) and so I didn't attempt it.



Perhaps I'm missing something about how $n_0$ was chosen such that $mu(E_{n_{0}}) <infty$...





Original Text:



The main part of my question comes from here:



enter image description here



Theorem 1.26:



enter image description here



And finally, Lemma 1.7:



enter image description here





Hopefully someone can shed some light!










share|cite|improve this question




















  • 2




    If $x$is in all but finitely many $E_{n+n_0}$, then it is also in all but finitely many $E_n$: the only sets $x$ is not in are the finitely many $E_i$ for $i ge n_0$, plus at most finitely many $E_j$ for j< n_0$. The converse is easier. Can you try to apply similar logic to the limsup part?
    – Mike Earnest
    Jan 4 at 1:01










  • Hey @MikeEarnest, I agree that $xinliminflimits_{nrightarrowinfty}E_{n+n_0}$ means that $x$ is not in finitely many $E_i$ for $igeq n_0$ and not in at most finitely many $E_j$ for $j < n_0$. But my problem is with the converse as $xinliminflimits_{nrightarrowinfty}E_{n}$ means that $x$ is not in finitely many $E_i$ for $ileq n_{0}' in mathbb{N}$. But $n_{0}'$ could be less than $n_0$ which means that $liminflimits_{nrightarrowinfty}E_{n}$ may include elements in sets $E_j$ for $n_{0}' leq j leq n_{0}$ .
    – Darius
    Jan 4 at 1:56










  • $I_{lim sup A_n}=lim sup I_{A_n}$ and $I_{lim inf A_n}=lim inf I_{A_n}$ so this reduces to elementary facts about limsup and liminf of sequences of real numbers. [Def: $I_A (x)=1$ if $x in A$, $0$ otherwise].
    – Kavi Rama Murthy
    Jan 4 at 6:20












  • @MikeEarnest, I just realized I was being a dummy! Thanks for your help!
    – Darius
    2 days ago
















1














I dropped the $nrightarrowinfty$ in the title as it was exceeding the character limit.



In the book I'm currently reading, the author claims that $liminflimits_{nrightarrowinfty} E_{n_0+n} = liminflimits_{nrightarrowinfty} E_{n}$ and $limsuplimits_{nrightarrowinfty} E_{n_0+n} = limsuplimits_{nrightarrowinfty} E_{n}$ where $mu(E_{n_{0}}) < infty$ and $E_n$ is a decreasing sequence? ($mu$ is an arbitrary measure!)





My attempt at a proof is as follows:



Let $x in liminflimits_{nrightarrowinfty} E_{n_0+n} = bigcup_{ninmathbb{N}}bigcap_{kgeq n}E_{n_0+n}$



Then $x in bigcap_{kgeq n}E_{n_0+n_{0}'}$ for some $n_{0}' in mathbb{N}$



But $bigcap_{kgeq n}E_{n_0+n_{0}'} subset bigcup_{ninmathbb{N}}bigcap_{kgeq n}E_{n} = liminflimits_{nrightarrowinfty} E_{n}$ and so $liminflimits_{nrightarrowinfty} E_{n_0+n} subset liminflimits_{nrightarrowinfty} E_{n}$.



I however couldn't prove the reverse inclusion in my attempt below:



Let $x in liminflimits_{nrightarrowinfty} E_{n} = bigcup_{ninmathbb{N}}bigcap_{kgeq n}E_{n}$



Then $x in bigcap_{kgeq n}E_{n_{0}''}$ for some $n_{0}'' in mathbb{N}$



I then realized it's not guaranteed that $n_{0}'' = n_{0}$ and so we may be "missing" some elements in $bigcup_{ninmathbb{N}}bigcap_{kgeq n}E_{n_0+n}$ from $bigcup_{ninmathbb{N}}bigcap_{kgeq n}E_{n}$.



I suspect the proof for $limsuplimits_{nrightarrowinfty} E_{n_0+n} = limsuplimits_{nrightarrowinfty} E_{n}$ is similar and would also fall apart at the same point as my proof for $liminflimits_{nrightarrowinfty} E_{n_0+n} = liminflimits_{nrightarrowinfty} E_{n}$ ($n_{0} neq n_{0}''$) and so I didn't attempt it.



Perhaps I'm missing something about how $n_0$ was chosen such that $mu(E_{n_{0}}) <infty$...





Original Text:



The main part of my question comes from here:



enter image description here



Theorem 1.26:



enter image description here



And finally, Lemma 1.7:



enter image description here





Hopefully someone can shed some light!










share|cite|improve this question




















  • 2




    If $x$is in all but finitely many $E_{n+n_0}$, then it is also in all but finitely many $E_n$: the only sets $x$ is not in are the finitely many $E_i$ for $i ge n_0$, plus at most finitely many $E_j$ for j< n_0$. The converse is easier. Can you try to apply similar logic to the limsup part?
    – Mike Earnest
    Jan 4 at 1:01










  • Hey @MikeEarnest, I agree that $xinliminflimits_{nrightarrowinfty}E_{n+n_0}$ means that $x$ is not in finitely many $E_i$ for $igeq n_0$ and not in at most finitely many $E_j$ for $j < n_0$. But my problem is with the converse as $xinliminflimits_{nrightarrowinfty}E_{n}$ means that $x$ is not in finitely many $E_i$ for $ileq n_{0}' in mathbb{N}$. But $n_{0}'$ could be less than $n_0$ which means that $liminflimits_{nrightarrowinfty}E_{n}$ may include elements in sets $E_j$ for $n_{0}' leq j leq n_{0}$ .
    – Darius
    Jan 4 at 1:56










  • $I_{lim sup A_n}=lim sup I_{A_n}$ and $I_{lim inf A_n}=lim inf I_{A_n}$ so this reduces to elementary facts about limsup and liminf of sequences of real numbers. [Def: $I_A (x)=1$ if $x in A$, $0$ otherwise].
    – Kavi Rama Murthy
    Jan 4 at 6:20












  • @MikeEarnest, I just realized I was being a dummy! Thanks for your help!
    – Darius
    2 days ago














1












1








1







I dropped the $nrightarrowinfty$ in the title as it was exceeding the character limit.



In the book I'm currently reading, the author claims that $liminflimits_{nrightarrowinfty} E_{n_0+n} = liminflimits_{nrightarrowinfty} E_{n}$ and $limsuplimits_{nrightarrowinfty} E_{n_0+n} = limsuplimits_{nrightarrowinfty} E_{n}$ where $mu(E_{n_{0}}) < infty$ and $E_n$ is a decreasing sequence? ($mu$ is an arbitrary measure!)





My attempt at a proof is as follows:



Let $x in liminflimits_{nrightarrowinfty} E_{n_0+n} = bigcup_{ninmathbb{N}}bigcap_{kgeq n}E_{n_0+n}$



Then $x in bigcap_{kgeq n}E_{n_0+n_{0}'}$ for some $n_{0}' in mathbb{N}$



But $bigcap_{kgeq n}E_{n_0+n_{0}'} subset bigcup_{ninmathbb{N}}bigcap_{kgeq n}E_{n} = liminflimits_{nrightarrowinfty} E_{n}$ and so $liminflimits_{nrightarrowinfty} E_{n_0+n} subset liminflimits_{nrightarrowinfty} E_{n}$.



I however couldn't prove the reverse inclusion in my attempt below:



Let $x in liminflimits_{nrightarrowinfty} E_{n} = bigcup_{ninmathbb{N}}bigcap_{kgeq n}E_{n}$



Then $x in bigcap_{kgeq n}E_{n_{0}''}$ for some $n_{0}'' in mathbb{N}$



I then realized it's not guaranteed that $n_{0}'' = n_{0}$ and so we may be "missing" some elements in $bigcup_{ninmathbb{N}}bigcap_{kgeq n}E_{n_0+n}$ from $bigcup_{ninmathbb{N}}bigcap_{kgeq n}E_{n}$.



I suspect the proof for $limsuplimits_{nrightarrowinfty} E_{n_0+n} = limsuplimits_{nrightarrowinfty} E_{n}$ is similar and would also fall apart at the same point as my proof for $liminflimits_{nrightarrowinfty} E_{n_0+n} = liminflimits_{nrightarrowinfty} E_{n}$ ($n_{0} neq n_{0}''$) and so I didn't attempt it.



Perhaps I'm missing something about how $n_0$ was chosen such that $mu(E_{n_{0}}) <infty$...





Original Text:



The main part of my question comes from here:



enter image description here



Theorem 1.26:



enter image description here



And finally, Lemma 1.7:



enter image description here





Hopefully someone can shed some light!










share|cite|improve this question















I dropped the $nrightarrowinfty$ in the title as it was exceeding the character limit.



In the book I'm currently reading, the author claims that $liminflimits_{nrightarrowinfty} E_{n_0+n} = liminflimits_{nrightarrowinfty} E_{n}$ and $limsuplimits_{nrightarrowinfty} E_{n_0+n} = limsuplimits_{nrightarrowinfty} E_{n}$ where $mu(E_{n_{0}}) < infty$ and $E_n$ is a decreasing sequence? ($mu$ is an arbitrary measure!)





My attempt at a proof is as follows:



Let $x in liminflimits_{nrightarrowinfty} E_{n_0+n} = bigcup_{ninmathbb{N}}bigcap_{kgeq n}E_{n_0+n}$



Then $x in bigcap_{kgeq n}E_{n_0+n_{0}'}$ for some $n_{0}' in mathbb{N}$



But $bigcap_{kgeq n}E_{n_0+n_{0}'} subset bigcup_{ninmathbb{N}}bigcap_{kgeq n}E_{n} = liminflimits_{nrightarrowinfty} E_{n}$ and so $liminflimits_{nrightarrowinfty} E_{n_0+n} subset liminflimits_{nrightarrowinfty} E_{n}$.



I however couldn't prove the reverse inclusion in my attempt below:



Let $x in liminflimits_{nrightarrowinfty} E_{n} = bigcup_{ninmathbb{N}}bigcap_{kgeq n}E_{n}$



Then $x in bigcap_{kgeq n}E_{n_{0}''}$ for some $n_{0}'' in mathbb{N}$



I then realized it's not guaranteed that $n_{0}'' = n_{0}$ and so we may be "missing" some elements in $bigcup_{ninmathbb{N}}bigcap_{kgeq n}E_{n_0+n}$ from $bigcup_{ninmathbb{N}}bigcap_{kgeq n}E_{n}$.



I suspect the proof for $limsuplimits_{nrightarrowinfty} E_{n_0+n} = limsuplimits_{nrightarrowinfty} E_{n}$ is similar and would also fall apart at the same point as my proof for $liminflimits_{nrightarrowinfty} E_{n_0+n} = liminflimits_{nrightarrowinfty} E_{n}$ ($n_{0} neq n_{0}''$) and so I didn't attempt it.



Perhaps I'm missing something about how $n_0$ was chosen such that $mu(E_{n_{0}}) <infty$...





Original Text:



The main part of my question comes from here:



enter image description here



Theorem 1.26:



enter image description here



And finally, Lemma 1.7:



enter image description here





Hopefully someone can shed some light!







real-analysis measure-theory elementary-set-theory limsup-and-liminf






share|cite|improve this question















share|cite|improve this question













share|cite|improve this question




share|cite|improve this question








edited Jan 4 at 0:38

























asked Jan 4 at 0:18









Darius

878




878








  • 2




    If $x$is in all but finitely many $E_{n+n_0}$, then it is also in all but finitely many $E_n$: the only sets $x$ is not in are the finitely many $E_i$ for $i ge n_0$, plus at most finitely many $E_j$ for j< n_0$. The converse is easier. Can you try to apply similar logic to the limsup part?
    – Mike Earnest
    Jan 4 at 1:01










  • Hey @MikeEarnest, I agree that $xinliminflimits_{nrightarrowinfty}E_{n+n_0}$ means that $x$ is not in finitely many $E_i$ for $igeq n_0$ and not in at most finitely many $E_j$ for $j < n_0$. But my problem is with the converse as $xinliminflimits_{nrightarrowinfty}E_{n}$ means that $x$ is not in finitely many $E_i$ for $ileq n_{0}' in mathbb{N}$. But $n_{0}'$ could be less than $n_0$ which means that $liminflimits_{nrightarrowinfty}E_{n}$ may include elements in sets $E_j$ for $n_{0}' leq j leq n_{0}$ .
    – Darius
    Jan 4 at 1:56










  • $I_{lim sup A_n}=lim sup I_{A_n}$ and $I_{lim inf A_n}=lim inf I_{A_n}$ so this reduces to elementary facts about limsup and liminf of sequences of real numbers. [Def: $I_A (x)=1$ if $x in A$, $0$ otherwise].
    – Kavi Rama Murthy
    Jan 4 at 6:20












  • @MikeEarnest, I just realized I was being a dummy! Thanks for your help!
    – Darius
    2 days ago














  • 2




    If $x$is in all but finitely many $E_{n+n_0}$, then it is also in all but finitely many $E_n$: the only sets $x$ is not in are the finitely many $E_i$ for $i ge n_0$, plus at most finitely many $E_j$ for j< n_0$. The converse is easier. Can you try to apply similar logic to the limsup part?
    – Mike Earnest
    Jan 4 at 1:01










  • Hey @MikeEarnest, I agree that $xinliminflimits_{nrightarrowinfty}E_{n+n_0}$ means that $x$ is not in finitely many $E_i$ for $igeq n_0$ and not in at most finitely many $E_j$ for $j < n_0$. But my problem is with the converse as $xinliminflimits_{nrightarrowinfty}E_{n}$ means that $x$ is not in finitely many $E_i$ for $ileq n_{0}' in mathbb{N}$. But $n_{0}'$ could be less than $n_0$ which means that $liminflimits_{nrightarrowinfty}E_{n}$ may include elements in sets $E_j$ for $n_{0}' leq j leq n_{0}$ .
    – Darius
    Jan 4 at 1:56










  • $I_{lim sup A_n}=lim sup I_{A_n}$ and $I_{lim inf A_n}=lim inf I_{A_n}$ so this reduces to elementary facts about limsup and liminf of sequences of real numbers. [Def: $I_A (x)=1$ if $x in A$, $0$ otherwise].
    – Kavi Rama Murthy
    Jan 4 at 6:20












  • @MikeEarnest, I just realized I was being a dummy! Thanks for your help!
    – Darius
    2 days ago








2




2




If $x$is in all but finitely many $E_{n+n_0}$, then it is also in all but finitely many $E_n$: the only sets $x$ is not in are the finitely many $E_i$ for $i ge n_0$, plus at most finitely many $E_j$ for j< n_0$. The converse is easier. Can you try to apply similar logic to the limsup part?
– Mike Earnest
Jan 4 at 1:01




If $x$is in all but finitely many $E_{n+n_0}$, then it is also in all but finitely many $E_n$: the only sets $x$ is not in are the finitely many $E_i$ for $i ge n_0$, plus at most finitely many $E_j$ for j< n_0$. The converse is easier. Can you try to apply similar logic to the limsup part?
– Mike Earnest
Jan 4 at 1:01












Hey @MikeEarnest, I agree that $xinliminflimits_{nrightarrowinfty}E_{n+n_0}$ means that $x$ is not in finitely many $E_i$ for $igeq n_0$ and not in at most finitely many $E_j$ for $j < n_0$. But my problem is with the converse as $xinliminflimits_{nrightarrowinfty}E_{n}$ means that $x$ is not in finitely many $E_i$ for $ileq n_{0}' in mathbb{N}$. But $n_{0}'$ could be less than $n_0$ which means that $liminflimits_{nrightarrowinfty}E_{n}$ may include elements in sets $E_j$ for $n_{0}' leq j leq n_{0}$ .
– Darius
Jan 4 at 1:56




Hey @MikeEarnest, I agree that $xinliminflimits_{nrightarrowinfty}E_{n+n_0}$ means that $x$ is not in finitely many $E_i$ for $igeq n_0$ and not in at most finitely many $E_j$ for $j < n_0$. But my problem is with the converse as $xinliminflimits_{nrightarrowinfty}E_{n}$ means that $x$ is not in finitely many $E_i$ for $ileq n_{0}' in mathbb{N}$. But $n_{0}'$ could be less than $n_0$ which means that $liminflimits_{nrightarrowinfty}E_{n}$ may include elements in sets $E_j$ for $n_{0}' leq j leq n_{0}$ .
– Darius
Jan 4 at 1:56












$I_{lim sup A_n}=lim sup I_{A_n}$ and $I_{lim inf A_n}=lim inf I_{A_n}$ so this reduces to elementary facts about limsup and liminf of sequences of real numbers. [Def: $I_A (x)=1$ if $x in A$, $0$ otherwise].
– Kavi Rama Murthy
Jan 4 at 6:20






$I_{lim sup A_n}=lim sup I_{A_n}$ and $I_{lim inf A_n}=lim inf I_{A_n}$ so this reduces to elementary facts about limsup and liminf of sequences of real numbers. [Def: $I_A (x)=1$ if $x in A$, $0$ otherwise].
– Kavi Rama Murthy
Jan 4 at 6:20














@MikeEarnest, I just realized I was being a dummy! Thanks for your help!
– Darius
2 days ago




@MikeEarnest, I just realized I was being a dummy! Thanks for your help!
– Darius
2 days ago










1 Answer
1






active

oldest

votes


















2














Let $F_n = E_{n+n_0}$ for some fixed $n_0$ as in your text.



Then $liminf_{n to infty} F_n$ is the set of all $x$ that are in all but finitely many $F_n$. But then $x$ is also in all but finitely many $E_n$ as $x$ can only miss the sets $E_0, ldots E_{n_0-1}$, i.e. finitely many. And if $x$ is in all but finitely many $E_n$ the same holds for the $F_n$ as we only throw some sets away. So the lemma indeed immediately applies that the liminfs of these shifted sequences of sets are the same. Similar reasoning applies to the limsups. Note that the $E_n$ need not be decreasing, they can be any sets.






share|cite|improve this answer























    Your Answer





    StackExchange.ifUsing("editor", function () {
    return StackExchange.using("mathjaxEditing", function () {
    StackExchange.MarkdownEditor.creationCallbacks.add(function (editor, postfix) {
    StackExchange.mathjaxEditing.prepareWmdForMathJax(editor, postfix, [["$", "$"], ["\\(","\\)"]]);
    });
    });
    }, "mathjax-editing");

    StackExchange.ready(function() {
    var channelOptions = {
    tags: "".split(" "),
    id: "69"
    };
    initTagRenderer("".split(" "), "".split(" "), channelOptions);

    StackExchange.using("externalEditor", function() {
    // Have to fire editor after snippets, if snippets enabled
    if (StackExchange.settings.snippets.snippetsEnabled) {
    StackExchange.using("snippets", function() {
    createEditor();
    });
    }
    else {
    createEditor();
    }
    });

    function createEditor() {
    StackExchange.prepareEditor({
    heartbeatType: 'answer',
    autoActivateHeartbeat: false,
    convertImagesToLinks: true,
    noModals: true,
    showLowRepImageUploadWarning: true,
    reputationToPostImages: 10,
    bindNavPrevention: true,
    postfix: "",
    imageUploader: {
    brandingHtml: "Powered by u003ca class="icon-imgur-white" href="https://imgur.com/"u003eu003c/au003e",
    contentPolicyHtml: "User contributions licensed under u003ca href="https://creativecommons.org/licenses/by-sa/3.0/"u003ecc by-sa 3.0 with attribution requiredu003c/au003e u003ca href="https://stackoverflow.com/legal/content-policy"u003e(content policy)u003c/au003e",
    allowUrls: true
    },
    noCode: true, onDemand: true,
    discardSelector: ".discard-answer"
    ,immediatelyShowMarkdownHelp:true
    });


    }
    });














    draft saved

    draft discarded


















    StackExchange.ready(
    function () {
    StackExchange.openid.initPostLogin('.new-post-login', 'https%3a%2f%2fmath.stackexchange.com%2fquestions%2f3061180%2fliminf-e-n-0n-liminf-limits-e-n-and-limsup-e-n-0n-limsup-e-n%23new-answer', 'question_page');
    }
    );

    Post as a guest















    Required, but never shown

























    1 Answer
    1






    active

    oldest

    votes








    1 Answer
    1






    active

    oldest

    votes









    active

    oldest

    votes






    active

    oldest

    votes









    2














    Let $F_n = E_{n+n_0}$ for some fixed $n_0$ as in your text.



    Then $liminf_{n to infty} F_n$ is the set of all $x$ that are in all but finitely many $F_n$. But then $x$ is also in all but finitely many $E_n$ as $x$ can only miss the sets $E_0, ldots E_{n_0-1}$, i.e. finitely many. And if $x$ is in all but finitely many $E_n$ the same holds for the $F_n$ as we only throw some sets away. So the lemma indeed immediately applies that the liminfs of these shifted sequences of sets are the same. Similar reasoning applies to the limsups. Note that the $E_n$ need not be decreasing, they can be any sets.






    share|cite|improve this answer




























      2














      Let $F_n = E_{n+n_0}$ for some fixed $n_0$ as in your text.



      Then $liminf_{n to infty} F_n$ is the set of all $x$ that are in all but finitely many $F_n$. But then $x$ is also in all but finitely many $E_n$ as $x$ can only miss the sets $E_0, ldots E_{n_0-1}$, i.e. finitely many. And if $x$ is in all but finitely many $E_n$ the same holds for the $F_n$ as we only throw some sets away. So the lemma indeed immediately applies that the liminfs of these shifted sequences of sets are the same. Similar reasoning applies to the limsups. Note that the $E_n$ need not be decreasing, they can be any sets.






      share|cite|improve this answer


























        2












        2








        2






        Let $F_n = E_{n+n_0}$ for some fixed $n_0$ as in your text.



        Then $liminf_{n to infty} F_n$ is the set of all $x$ that are in all but finitely many $F_n$. But then $x$ is also in all but finitely many $E_n$ as $x$ can only miss the sets $E_0, ldots E_{n_0-1}$, i.e. finitely many. And if $x$ is in all but finitely many $E_n$ the same holds for the $F_n$ as we only throw some sets away. So the lemma indeed immediately applies that the liminfs of these shifted sequences of sets are the same. Similar reasoning applies to the limsups. Note that the $E_n$ need not be decreasing, they can be any sets.






        share|cite|improve this answer














        Let $F_n = E_{n+n_0}$ for some fixed $n_0$ as in your text.



        Then $liminf_{n to infty} F_n$ is the set of all $x$ that are in all but finitely many $F_n$. But then $x$ is also in all but finitely many $E_n$ as $x$ can only miss the sets $E_0, ldots E_{n_0-1}$, i.e. finitely many. And if $x$ is in all but finitely many $E_n$ the same holds for the $F_n$ as we only throw some sets away. So the lemma indeed immediately applies that the liminfs of these shifted sequences of sets are the same. Similar reasoning applies to the limsups. Note that the $E_n$ need not be decreasing, they can be any sets.







        share|cite|improve this answer














        share|cite|improve this answer



        share|cite|improve this answer








        edited 2 days ago

























        answered Jan 4 at 5:54









        Henno Brandsma

        105k347114




        105k347114






























            draft saved

            draft discarded




















































            Thanks for contributing an answer to Mathematics Stack Exchange!


            • Please be sure to answer the question. Provide details and share your research!

            But avoid



            • Asking for help, clarification, or responding to other answers.

            • Making statements based on opinion; back them up with references or personal experience.


            Use MathJax to format equations. MathJax reference.


            To learn more, see our tips on writing great answers.





            Some of your past answers have not been well-received, and you're in danger of being blocked from answering.


            Please pay close attention to the following guidance:


            • Please be sure to answer the question. Provide details and share your research!

            But avoid



            • Asking for help, clarification, or responding to other answers.

            • Making statements based on opinion; back them up with references or personal experience.


            To learn more, see our tips on writing great answers.




            draft saved


            draft discarded














            StackExchange.ready(
            function () {
            StackExchange.openid.initPostLogin('.new-post-login', 'https%3a%2f%2fmath.stackexchange.com%2fquestions%2f3061180%2fliminf-e-n-0n-liminf-limits-e-n-and-limsup-e-n-0n-limsup-e-n%23new-answer', 'question_page');
            }
            );

            Post as a guest















            Required, but never shown





















































            Required, but never shown














            Required, but never shown












            Required, but never shown







            Required, but never shown

































            Required, but never shown














            Required, but never shown












            Required, but never shown







            Required, but never shown







            Popular posts from this blog

            An IMO inspired problem

            Management

            Has there ever been an instance of an active nuclear power plant within or near a war zone?